Inscription / Connexion Nouveau Sujet
Niveau Maths sup
Partager :

logique

Posté par
ferenc
25-12-11 à 12:08

Bonjour,

Q1) quelle est la différence entre: \forall x\in\R,f(x)=x et  f(x)=x,\forall x\in\R ?
Moi je dirais que que dans le premier cas, on a que f dépend de x et dans le deuxième cas c'est x qui dépend de f(x), non ?

Q2) Soit la proposition: \forall y\in\R,\exists x\in\R: y=f(x) a t-on que x dépend de y ?
Je pense que oui, malgré le fait que si y=f(x) on a que y dépend de x

Q3) Si \exists x,\forall y,y\leq x, a t-on que x est fixé à priori ?
En gros si j'ai cette propriété, puis-je dire: Soit x tel que \forall y,y\leq x ?

Q4) Pour la même proposition que Q3) a-ton y qui dépend de x ou bien x qui dépend de y?

Moi j'aurais dis que c'est x qui dépend de y puisque en fait c'est la définition d'un ensemble majoré, donc on ne peut fixé le x que en fonction du y

Q5 (et dernière))
Pour moi, si \exists x,\forall y alors \forall y,\exists x or j'ai lu plusieurs fois que cette implication été fausse ! (Il va de soi que la réciproque est fausse), qu'en pensez vous ?

Posté par
ferenc
re : logique 25-12-11 à 12:10

Merci et joyeux noel !

Posté par
Surb
re : logique 25-12-11 à 14:09

Bonjour,
Tout ne se résume pas à des concepts de dépendance....

Q1) il n'y a aucune différence entre les deux, de plus ça n'a pas de sens de faire dépendre f de x (à moins que tu travailles sur un espace de fonction ce qui m'étonnerait) et de faire dépendre x de f (dans ce cas)...
Ca veut simplement dire que la fonction f est définie par
f:\mathbb{R}\to\mathbb{R}:x \to x

Q2) Ceci veut simplement dire que f est surjective sur \mathbb{R}. Et donc oui x va dépendre de y (à noter que ce x n'est pas forcément unique!). Dire que f(x) dépend de x est un pléonasme.

Q3,4) Si un ensemble est majoré alors par définition il existe un nombre qui est plus grand que tout les membres de cet ensemble. Tu peux donc dire soit x un majorant de l'ensemble. Comme il est plus grand que tous les éléments de l'ensemble on a effectivement y \leq x pour tout y dans l'ensemble. Il est évident que le choix de x dépend entièrement des éléments y qu'il y a dans l'ensemble.

Q5) c'est effectivement faux! Prenons un exemple simple:
- \exists un homme x tel que \forall homme y, x est strictement plus riche que y (on prendra l'homme le plus riche du monde, en supposant qu'il n'y en a pas deux exæquo).
- \forall homme y \exists un homme x qui est strictement plus riche, ceci est faux lorsqu'on choisi y comme étant l'homme le plus riche du monde.

Posté par
ferenc
re : logique 25-12-11 à 15:05

merci pour toute vos réponses. j'ai donc quelques précisions à vous demander si vous avez le temps de me les fournir.

Pour Q3) et Q4) je vais donc vous prendre un autre exemple.

Soit f:I\to\R

\bullet Si f continue sur I, on a que \forall y\in I,\lim_{x\to y}f(x)=f(y), ce qui signifie que:
\forall y\in I,\forall \epsilon>0,\exists\delta>0:\forall x\in I,|x-y|<\delta\Rightarrow |f(x)-f(y)|<\epsilon
On est donc d'accord que \delta dépend à priori de y et de \epsilon

Q1) Mais qu'en est t-il de x ?, x est indépendante ? ou bien x dépend de \delta (et donc de y et \epsilon) ou bien juste de y ?

\bullet Supposons f uniformément continue. Ainsi:
\forall \epsilon>0,\exists\delta>0:\forall x,y\in I,|x-y|\leq\delta\Rightarrow |f(x)-f(y)|<\epsilon
On est d'accord que \delta ne dépend que de \epsilon

Q2) mais qu'en est t-il de x et y ?

\bullet Si par exemple elle n'est pas continue en y\in I alors \exists \epsilon>0,\forall \delta>0:\exists x\in I, |x-y|<\delta\wedge |f(x)-f(y)|\geq\epsilon

Q3) Dans ce contexte, \epsilon dépend de quoi ? \delta dépend de quoi ? et x dépend de quoi ?


Pour Q5)
Je ne suis pas convaincu par votre exemple car ce qui me gène, c'est que si \exists x un homme telle que \forall y hommes, x est strictement plus riche que y, puisque x est un homme, alors x est compris dans y, et puisque x est l'homme le plus riche, en prenant y=x votre propriété est fausse ! il est donc evident que la réciproque le sera aussi !
En revanche si vous prenant y\in \{hommes\}\backslash\{x\}, la propriété \exists,\forall sera juste, et la réciproque aussi.. héhé

Voyez plutôt:
\bullet Si \exists une clé telle que \forall les serrure des portes de l'établissement, cette serrure est ouverte par cette clé, alors nécessairement \forall les serrures de l'établissement, \exists une clé qui l'ouvre (ce qu'on appelle un passe-partout)

Mais par exemple, si \forall serrure de l'établissement, \exists une clé qui l'ouvre, il n'existe pas forcément une clé qui ouvre toute les serrures (non existence d'un passe partout)

Donc je m'obstine à croire que \exists x,\forall y\Rightarrow \forall y,\exists x, mais la réciproque est bien sûr fausse !

Merci d'avoir pris le temps de lire ce message (et si possible de me répondre ), ce serait un beau cadeau de noël

joyeux noël

Posté par
Bachstelze
re : logique 25-12-11 à 15:24

Déjà, il faudrait préciser ce que tu entends par "dépend"... Donc je suppose que par exemple si je dis:

x , y , x+y = 0

tu dis que y "dépend" de x, parce que pour deux valeurs différentes de y, la valeur de x qui satisfait x+y = 0 pourra être différente (et dans ce cas précis, le sera forcément, mais pas toujours).

1) x ne "dépend" de rien du tout, puisque tu dois de toute façon considérer toutes les valeurs de x (dans I).

2) De même.

3) dépend de f, puisque pour chaque fonction, il y aura (au moins) une valeur de qui convient, mais qu'une valeur qui convient pour une fonction ne conviendra pas forcément pour une autre.

Posté par
Bachstelze
re : logique 25-12-11 à 15:25

Pardon, pour deux valuers différentes de x, la valeur de y qui satisfait x+y = 0 pourra être différente.

Posté par
Bachstelze
re : logique 25-12-11 à 15:29

Et par pitié, n'utilise pas les quantificateurs dans une phrase en français ! Une place pour chaque chose et chaque chose à sa place. Si je suis prof de maths et qu'un élève m'écrit un truc style " réel x, un réel y tel que x+y = 0", je lui mets 0. (C'est pour ça que je suis pas prof de amths. )

Posté par
ferenc
re : logique 25-12-11 à 15:37

\bullet pour 1) et 2) c'est très clair !!!

\bullet Pour 3) je pense que c'est clair, mais aura t-on quand même x qui dépendra de \delta (je pense que oui et c'est évident, mais c'est juste pour avoir confirmation d'un EXPERT ^^)
Cependant une chose me chiffonne, Si \exists \epsilon,\forall\delta\Rightarrow\forall\delta,\exists\epsilon (Je n'ai pas encore confirmation de cela, c'est d'ailleurs l'objet de Q5), mais je n'ai toujours pas de contre exemple qui me dise le contraire), on a que \epsilon dépend aussi de \delta
Mais je pense que c'est une conséquence indirecte du fait que \epsilon dépende de f car f dépend de x et x dépend à priori de \delta, qu'en pensez vous ?

Pour votre dernière remarque, je ne le fait jamais, c'était juste pour illustrer l'ordre des quantificateur !!!

Pour Q5) une idée ?

Posté par
Bachstelze
re : logique 25-12-11 à 15:56

Pour la 3, x dépend de , oui.

Pour la 5 l'implication est évidemment vraie.

Posté par
ferenc
re : logique 25-12-11 à 16:31

et bien merci Bachstelze !



Vous devez être membre accéder à ce service...

Pas encore inscrit ?

1 compte par personne, multi-compte interdit !

Ou identifiez-vous :


Rester sur la page

Inscription gratuite

Fiches en rapport

parmi 1675 fiches de maths

Désolé, votre version d'Internet Explorer est plus que périmée ! Merci de le mettre à jour ou de télécharger Firefox ou Google Chrome pour utiliser le site. Votre ordinateur vous remerciera !